Trying to find a flaw in my proof that there are more rearrangements of an infinite series than real numbersQuestion on Riemann's rearrangement theoremProof that the real numbers are countable: Help with why this is wrongHow to divide aleph numbersHow to make Riemann rearrangement?All sets of rational numbers are bigger than the set containing infinite integers - or are they?Bijection between $mathbbZ longmapsto mathbbR$Verification of this proof that the set of real numbers is uncountable.Proof that the set of Natural numbers are equal to the Real numbersRearrangement of Alternating Harmonic Series to be InfinityUsing union of countably infinite sets, I tried to prove that set of all real numbers in [0,1) is countable

Why do players in the past play much longer tournaments than today's top players?

Is it rude to tell recruiters I would only change jobs for a better salary?

Geometric interpretation of complex inner products

What would the EU do if an EU member declared war on another EU member?

Returning the argument of a function if the argument is not of the right type

Are there any double stars that I can actually see orbit each other?

A DVR algebra with weird automorphisms

Cops: The Hidden OEIS Substring

Email about missed connecting flight compensation 5 months after flight, is there a point?

Can I intentionally omit previous work experience or pretend it doesn't exist when applying for jobs?

If a specific mass of air is polluted, will the pollution stick with it?

Where or how can I find what interfaces an out of the box Apex class implements?

Is Arc Length always irrational between two rational points?

Would letting a multiclass character rebuild their character to be single-classed be game-breaking?

What is the difference between logical consistency and logical entailment in deductive logic?

As a DM, how to avoid unconscious metagaming when dealing with a high AC character?

Steampunk helicopter

<schwitz>, <zwinker> etc. Does German always use 2nd Person Singular Imperative verbs for emoticons? If so, why?

When did the Roman Empire fall according to contemporaries?

Is killing off one of my queer characters homophobic?

Can I play a first turn Simic Growth Chamber to have 3 mana available in the second turn?

How can I deal with a player trying to insert real-world mythology into my homebrew setting?

Wrapper in return method for test class

What would be the ideal melee weapon made of "Phase Metal"?



Trying to find a flaw in my proof that there are more rearrangements of an infinite series than real numbers


Question on Riemann's rearrangement theoremProof that the real numbers are countable: Help with why this is wrongHow to divide aleph numbersHow to make Riemann rearrangement?All sets of rational numbers are bigger than the set containing infinite integers - or are they?Bijection between $mathbbZ longmapsto mathbbR$Verification of this proof that the set of real numbers is uncountable.Proof that the set of Natural numbers are equal to the Real numbersRearrangement of Alternating Harmonic Series to be InfinityUsing union of countably infinite sets, I tried to prove that set of all real numbers in [0,1) is countable






.everyoneloves__top-leaderboard:empty,.everyoneloves__mid-leaderboard:empty,.everyoneloves__bot-mid-leaderboard:empty margin-bottom:0;








3












$begingroup$


So I had this thought that I was trying to prove as an exercise




Let $mathbbR$ be the set of real numbers and let $mathbbS$ be the set of all possible rearrangements of the alternating harmonic series. Prove that $|mathbbR| < |mathbbS|$




I thought I had a proof of this, but I then posted it to Reddit /r/math only to be downvoted and told the proof was wrong. The only comment I received was to "look at it from the other direction", but that confused me.



Here is my proof:




Two sets have the same cardinality iff there exists a bijection between them.



From the rearrangement theorem we can show that a the alternating harmonic series can converge to any real number via the following algorithm:



Start with $1$, if this is larger than the target number add the next negative term, otherwise add the next positive term. We create a mapping from the created rearrangement to the limit of this rearrangement. Notice that this maps to all real numbers.



Now take one of the series that we had, and switch the first two terms. This is a new rearrangement since it does not begin with $1$, so it should be mapped to a new real number. However all real numbers have already had a rearrangement mapped to them. As such we have two rearrangements pointing to a single real number, which means that our mapping is not a bijection.



As such there must be more rearrangements than real numbers.




Now I am not sure where my proof went wrong, so any help would be appreciated!










share|cite|improve this question









$endgroup$







  • 2




    $begingroup$
    That doesn't prove that there are more rearrangements. It proves that there are at least as many.
    $endgroup$
    – Matt Samuel
    8 hours ago






  • 1




    $begingroup$
    I can map the integers into themselves injectively but not bijectively (say $F(n) = 2n$), but that doesn't mean the there are more integers than there are integers. Or if you prefer, I can map the even integers onto the integers, but the odds are left over in the domain ... but you can't conclude the domain is bigger than the range. One map is not all maps!
    $endgroup$
    – Ned
    8 hours ago







  • 1




    $begingroup$
    The cardinality of the arrangements is at most that of $mathbb N^mathbb N $. I don't know if that's bigger than $c$, but I suspect it's not.
    $endgroup$
    – Matt Samuel
    8 hours ago










  • $begingroup$
    @Ned, I see what you mean, but then how does Cantor's diagonal proof work? Isn't that creating a map and then showing that there are more real numbers that have not yet been mapped to Naturals? Sorry for the basic question I know you're right it just has yet to "click" for me
    $endgroup$
    – wjmccann
    8 hours ago






  • 2




    $begingroup$
    @wjmccann The difference in the diagonal argument is it finds an additional element no matter what the map is. It gives you a method to construct a real number that an arbitrary map doesn't hit.
    $endgroup$
    – Matt Samuel
    8 hours ago

















3












$begingroup$


So I had this thought that I was trying to prove as an exercise




Let $mathbbR$ be the set of real numbers and let $mathbbS$ be the set of all possible rearrangements of the alternating harmonic series. Prove that $|mathbbR| < |mathbbS|$




I thought I had a proof of this, but I then posted it to Reddit /r/math only to be downvoted and told the proof was wrong. The only comment I received was to "look at it from the other direction", but that confused me.



Here is my proof:




Two sets have the same cardinality iff there exists a bijection between them.



From the rearrangement theorem we can show that a the alternating harmonic series can converge to any real number via the following algorithm:



Start with $1$, if this is larger than the target number add the next negative term, otherwise add the next positive term. We create a mapping from the created rearrangement to the limit of this rearrangement. Notice that this maps to all real numbers.



Now take one of the series that we had, and switch the first two terms. This is a new rearrangement since it does not begin with $1$, so it should be mapped to a new real number. However all real numbers have already had a rearrangement mapped to them. As such we have two rearrangements pointing to a single real number, which means that our mapping is not a bijection.



As such there must be more rearrangements than real numbers.




Now I am not sure where my proof went wrong, so any help would be appreciated!










share|cite|improve this question









$endgroup$







  • 2




    $begingroup$
    That doesn't prove that there are more rearrangements. It proves that there are at least as many.
    $endgroup$
    – Matt Samuel
    8 hours ago






  • 1




    $begingroup$
    I can map the integers into themselves injectively but not bijectively (say $F(n) = 2n$), but that doesn't mean the there are more integers than there are integers. Or if you prefer, I can map the even integers onto the integers, but the odds are left over in the domain ... but you can't conclude the domain is bigger than the range. One map is not all maps!
    $endgroup$
    – Ned
    8 hours ago







  • 1




    $begingroup$
    The cardinality of the arrangements is at most that of $mathbb N^mathbb N $. I don't know if that's bigger than $c$, but I suspect it's not.
    $endgroup$
    – Matt Samuel
    8 hours ago










  • $begingroup$
    @Ned, I see what you mean, but then how does Cantor's diagonal proof work? Isn't that creating a map and then showing that there are more real numbers that have not yet been mapped to Naturals? Sorry for the basic question I know you're right it just has yet to "click" for me
    $endgroup$
    – wjmccann
    8 hours ago






  • 2




    $begingroup$
    @wjmccann The difference in the diagonal argument is it finds an additional element no matter what the map is. It gives you a method to construct a real number that an arbitrary map doesn't hit.
    $endgroup$
    – Matt Samuel
    8 hours ago













3












3








3





$begingroup$


So I had this thought that I was trying to prove as an exercise




Let $mathbbR$ be the set of real numbers and let $mathbbS$ be the set of all possible rearrangements of the alternating harmonic series. Prove that $|mathbbR| < |mathbbS|$




I thought I had a proof of this, but I then posted it to Reddit /r/math only to be downvoted and told the proof was wrong. The only comment I received was to "look at it from the other direction", but that confused me.



Here is my proof:




Two sets have the same cardinality iff there exists a bijection between them.



From the rearrangement theorem we can show that a the alternating harmonic series can converge to any real number via the following algorithm:



Start with $1$, if this is larger than the target number add the next negative term, otherwise add the next positive term. We create a mapping from the created rearrangement to the limit of this rearrangement. Notice that this maps to all real numbers.



Now take one of the series that we had, and switch the first two terms. This is a new rearrangement since it does not begin with $1$, so it should be mapped to a new real number. However all real numbers have already had a rearrangement mapped to them. As such we have two rearrangements pointing to a single real number, which means that our mapping is not a bijection.



As such there must be more rearrangements than real numbers.




Now I am not sure where my proof went wrong, so any help would be appreciated!










share|cite|improve this question









$endgroup$




So I had this thought that I was trying to prove as an exercise




Let $mathbbR$ be the set of real numbers and let $mathbbS$ be the set of all possible rearrangements of the alternating harmonic series. Prove that $|mathbbR| < |mathbbS|$




I thought I had a proof of this, but I then posted it to Reddit /r/math only to be downvoted and told the proof was wrong. The only comment I received was to "look at it from the other direction", but that confused me.



Here is my proof:




Two sets have the same cardinality iff there exists a bijection between them.



From the rearrangement theorem we can show that a the alternating harmonic series can converge to any real number via the following algorithm:



Start with $1$, if this is larger than the target number add the next negative term, otherwise add the next positive term. We create a mapping from the created rearrangement to the limit of this rearrangement. Notice that this maps to all real numbers.



Now take one of the series that we had, and switch the first two terms. This is a new rearrangement since it does not begin with $1$, so it should be mapped to a new real number. However all real numbers have already had a rearrangement mapped to them. As such we have two rearrangements pointing to a single real number, which means that our mapping is not a bijection.



As such there must be more rearrangements than real numbers.




Now I am not sure where my proof went wrong, so any help would be appreciated!







sequences-and-series general-topology elementary-set-theory real-numbers infinity






share|cite|improve this question













share|cite|improve this question











share|cite|improve this question




share|cite|improve this question










asked 8 hours ago









wjmccannwjmccann

7222 silver badges18 bronze badges




7222 silver badges18 bronze badges







  • 2




    $begingroup$
    That doesn't prove that there are more rearrangements. It proves that there are at least as many.
    $endgroup$
    – Matt Samuel
    8 hours ago






  • 1




    $begingroup$
    I can map the integers into themselves injectively but not bijectively (say $F(n) = 2n$), but that doesn't mean the there are more integers than there are integers. Or if you prefer, I can map the even integers onto the integers, but the odds are left over in the domain ... but you can't conclude the domain is bigger than the range. One map is not all maps!
    $endgroup$
    – Ned
    8 hours ago







  • 1




    $begingroup$
    The cardinality of the arrangements is at most that of $mathbb N^mathbb N $. I don't know if that's bigger than $c$, but I suspect it's not.
    $endgroup$
    – Matt Samuel
    8 hours ago










  • $begingroup$
    @Ned, I see what you mean, but then how does Cantor's diagonal proof work? Isn't that creating a map and then showing that there are more real numbers that have not yet been mapped to Naturals? Sorry for the basic question I know you're right it just has yet to "click" for me
    $endgroup$
    – wjmccann
    8 hours ago






  • 2




    $begingroup$
    @wjmccann The difference in the diagonal argument is it finds an additional element no matter what the map is. It gives you a method to construct a real number that an arbitrary map doesn't hit.
    $endgroup$
    – Matt Samuel
    8 hours ago












  • 2




    $begingroup$
    That doesn't prove that there are more rearrangements. It proves that there are at least as many.
    $endgroup$
    – Matt Samuel
    8 hours ago






  • 1




    $begingroup$
    I can map the integers into themselves injectively but not bijectively (say $F(n) = 2n$), but that doesn't mean the there are more integers than there are integers. Or if you prefer, I can map the even integers onto the integers, but the odds are left over in the domain ... but you can't conclude the domain is bigger than the range. One map is not all maps!
    $endgroup$
    – Ned
    8 hours ago







  • 1




    $begingroup$
    The cardinality of the arrangements is at most that of $mathbb N^mathbb N $. I don't know if that's bigger than $c$, but I suspect it's not.
    $endgroup$
    – Matt Samuel
    8 hours ago










  • $begingroup$
    @Ned, I see what you mean, but then how does Cantor's diagonal proof work? Isn't that creating a map and then showing that there are more real numbers that have not yet been mapped to Naturals? Sorry for the basic question I know you're right it just has yet to "click" for me
    $endgroup$
    – wjmccann
    8 hours ago






  • 2




    $begingroup$
    @wjmccann The difference in the diagonal argument is it finds an additional element no matter what the map is. It gives you a method to construct a real number that an arbitrary map doesn't hit.
    $endgroup$
    – Matt Samuel
    8 hours ago







2




2




$begingroup$
That doesn't prove that there are more rearrangements. It proves that there are at least as many.
$endgroup$
– Matt Samuel
8 hours ago




$begingroup$
That doesn't prove that there are more rearrangements. It proves that there are at least as many.
$endgroup$
– Matt Samuel
8 hours ago




1




1




$begingroup$
I can map the integers into themselves injectively but not bijectively (say $F(n) = 2n$), but that doesn't mean the there are more integers than there are integers. Or if you prefer, I can map the even integers onto the integers, but the odds are left over in the domain ... but you can't conclude the domain is bigger than the range. One map is not all maps!
$endgroup$
– Ned
8 hours ago





$begingroup$
I can map the integers into themselves injectively but not bijectively (say $F(n) = 2n$), but that doesn't mean the there are more integers than there are integers. Or if you prefer, I can map the even integers onto the integers, but the odds are left over in the domain ... but you can't conclude the domain is bigger than the range. One map is not all maps!
$endgroup$
– Ned
8 hours ago





1




1




$begingroup$
The cardinality of the arrangements is at most that of $mathbb N^mathbb N $. I don't know if that's bigger than $c$, but I suspect it's not.
$endgroup$
– Matt Samuel
8 hours ago




$begingroup$
The cardinality of the arrangements is at most that of $mathbb N^mathbb N $. I don't know if that's bigger than $c$, but I suspect it's not.
$endgroup$
– Matt Samuel
8 hours ago












$begingroup$
@Ned, I see what you mean, but then how does Cantor's diagonal proof work? Isn't that creating a map and then showing that there are more real numbers that have not yet been mapped to Naturals? Sorry for the basic question I know you're right it just has yet to "click" for me
$endgroup$
– wjmccann
8 hours ago




$begingroup$
@Ned, I see what you mean, but then how does Cantor's diagonal proof work? Isn't that creating a map and then showing that there are more real numbers that have not yet been mapped to Naturals? Sorry for the basic question I know you're right it just has yet to "click" for me
$endgroup$
– wjmccann
8 hours ago




2




2




$begingroup$
@wjmccann The difference in the diagonal argument is it finds an additional element no matter what the map is. It gives you a method to construct a real number that an arbitrary map doesn't hit.
$endgroup$
– Matt Samuel
8 hours ago




$begingroup$
@wjmccann The difference in the diagonal argument is it finds an additional element no matter what the map is. It gives you a method to construct a real number that an arbitrary map doesn't hit.
$endgroup$
– Matt Samuel
8 hours ago










4 Answers
4






active

oldest

votes


















3












$begingroup$

While you showed $|mathbbR| leq |mathbbS|$ properly by constructing a surjection, the problem with your proof of $|mathbbR| neq |mathbbS|$ is that it merely shows that the particular kind of map you are trying to construct cannot be a bijection. For the proof to be valid, it would need to show that any map from $mathbbS$ to $mathbbR$ is not a bijection.






share|cite|improve this answer









$endgroup$




















    3












    $begingroup$

    As explained in the comments, what you've done is whip up a non-injective surjection from $mathbbS$ to $mathbbR$. This alone doesn't prove the statement in question, however - just because one surjection isn't injective doesn't mean that there isn't some other surjection which is injective.



    This is a fundamental difference between the infinite and finite cases. If $A$ and $B$ are finite sets and $f:Arightarrow B$ is a non-injective surjection, then $A$ does in fact have greater cardinality than $B$. But this fails for infinite sets: consider for example the map from naturals to naturals given by $$xmapsto [xover 2]$$ (where "$[cdot]$" is the floor function). This is a non-injective surjection; should we conclude that $vertmathbbNvert>vertmathbbNvert$? You can easily whip up other examples to drive the point home.



    Indeed, $mathbbR$ and $mathbbS$ have the same cardinality. Matt Samuel's comment has suggested an explanation: that the set $mathbbN^mathbbN$ of maps from naturals to naturals - or equivalently, infinite sequences of natural numbers - has cardinality at least that of $mathbbS$, and so we'll be done if we can find an injection from $mathbbN^mathbbN$ to $mathbbR$. We can do this by "coding into the binary expansion" - think about how the real number $$0.colorred001colorred00001colorred0001colorred01...$$ corresponds to the sequence $$2,4,3,1,....$$ Do you see how to turn this into an honest-to-goodness injection?




    Incidentally, while it doesn't make a difference in the context of the axiom of choice (which tells us that given a surjection $Arightarrow B$ we can find an injection $Brightarrow A$), it will ultimately be better to think of cardinality in terms of injections rather than surjections: $vert Avertgevert Bvert$ iff there is an injection $Brightarrow A$, not iff there is a surjection $Arightarrow B$. It's not immediately obvious that this is a better notion when choice fails, but it really does turn out to be the right one.






    share|cite|improve this answer









    $endgroup$












    • $begingroup$
      Funny how we both wrote similar answers. :P
      $endgroup$
      – Asaf Karagila
      7 hours ago










    • $begingroup$
      @AsafKaragila As is our way. (Beat me by two minutes ...)
      $endgroup$
      – Noah Schweber
      7 hours ago


















    2












    $begingroup$

    Consider the function $FcolonBbbNto N$ defined by: $$F(n)=begincases 0 & n=0\ n-1 & n>0endcases$$
    by your argument, since this function is not a bijection (clearly $F(0)=F(1)$ so it is not injective), $Bbb N$ is not the same size as $Bbb N$.




    What just happened? You've proved, as I did above, there is a surjective function which is not injective. But that doesn't prevents a different function to be a bijection.



    Cantor's diagonal works by showing that any function is not surjective, and therefore there are no bijections. But you've only showed that one function is not injective, and concluded that there are no bijections.



    You have to remember: Infinite sets are very different from finite sets. Just because one function is a surjection which is not a bijection doesn't mean there a different one is not.



    Here's an injective function from the rearrangements into the reals:



    Let $fcolonBbbNto N$ be the permutation describing the rearrangement. Namely, $f(n)=k$ if and only if the $k$th summand is $frac1n$ (up to a sign, if you prefer). Map the rearrangement given by $f$ to the real number given by: $$sum_ninBbb Nfrac23^2^ncdot 3^f(n)$$



    This sum converges, and you can quite easily show that if $fneq g$, then there is some $n$ such that $f(n)neq g(n)$ and therefore one of the digits in the trenary expansion of the two reals is different, and since those are only $0$ or $2$ this means that the real numbers are different. Therefore the map from rearrangements into the real numbers given above is injective.



    On the other hand, you can easily show that there are at least $2^aleph_0$ rearrangements, but I will leave this to you as an exercise.






    share|cite|improve this answer









    $endgroup$




















      2












      $begingroup$

      You have shown the cardinality of the real numbers is a lower bound by constructing a surjective map. You can't conclude from the fact that it is not injective that there are more rearrangements, because in order to do that you'd need to be able to show that every map is not injective.



      In fact, the cardinality of the set of functions from $mathbb N$ to $mathbb N$ is an upper bound, and that is the same as that of the real numbers. So in fact there are equally many arrangements as real numbers.



      In Cantor's diagonal argument, we are given an arbitrary function from the naturals to the real numbers, and the argument provides an algorithm for finding a number that the function doesn't hit. This works because it's not just one function this works for, it is every function. Therefore no function from the naturals to the real numbers is surjective.






      share|cite|improve this answer











      $endgroup$















        Your Answer








        StackExchange.ready(function()
        var channelOptions =
        tags: "".split(" "),
        id: "69"
        ;
        initTagRenderer("".split(" "), "".split(" "), channelOptions);

        StackExchange.using("externalEditor", function()
        // Have to fire editor after snippets, if snippets enabled
        if (StackExchange.settings.snippets.snippetsEnabled)
        StackExchange.using("snippets", function()
        createEditor();
        );

        else
        createEditor();

        );

        function createEditor()
        StackExchange.prepareEditor(
        heartbeatType: 'answer',
        autoActivateHeartbeat: false,
        convertImagesToLinks: true,
        noModals: true,
        showLowRepImageUploadWarning: true,
        reputationToPostImages: 10,
        bindNavPrevention: true,
        postfix: "",
        imageUploader:
        brandingHtml: "Powered by u003ca class="icon-imgur-white" href="https://imgur.com/"u003eu003c/au003e",
        contentPolicyHtml: "User contributions licensed under u003ca href="https://creativecommons.org/licenses/by-sa/3.0/"u003ecc by-sa 3.0 with attribution requiredu003c/au003e u003ca href="https://stackoverflow.com/legal/content-policy"u003e(content policy)u003c/au003e",
        allowUrls: true
        ,
        noCode: true, onDemand: true,
        discardSelector: ".discard-answer"
        ,immediatelyShowMarkdownHelp:true
        );



        );













        draft saved

        draft discarded


















        StackExchange.ready(
        function ()
        StackExchange.openid.initPostLogin('.new-post-login', 'https%3a%2f%2fmath.stackexchange.com%2fquestions%2f3291289%2ftrying-to-find-a-flaw-in-my-proof-that-there-are-more-rearrangements-of-an-infin%23new-answer', 'question_page');

        );

        Post as a guest















        Required, but never shown

























        4 Answers
        4






        active

        oldest

        votes








        4 Answers
        4






        active

        oldest

        votes









        active

        oldest

        votes






        active

        oldest

        votes









        3












        $begingroup$

        While you showed $|mathbbR| leq |mathbbS|$ properly by constructing a surjection, the problem with your proof of $|mathbbR| neq |mathbbS|$ is that it merely shows that the particular kind of map you are trying to construct cannot be a bijection. For the proof to be valid, it would need to show that any map from $mathbbS$ to $mathbbR$ is not a bijection.






        share|cite|improve this answer









        $endgroup$

















          3












          $begingroup$

          While you showed $|mathbbR| leq |mathbbS|$ properly by constructing a surjection, the problem with your proof of $|mathbbR| neq |mathbbS|$ is that it merely shows that the particular kind of map you are trying to construct cannot be a bijection. For the proof to be valid, it would need to show that any map from $mathbbS$ to $mathbbR$ is not a bijection.






          share|cite|improve this answer









          $endgroup$















            3












            3








            3





            $begingroup$

            While you showed $|mathbbR| leq |mathbbS|$ properly by constructing a surjection, the problem with your proof of $|mathbbR| neq |mathbbS|$ is that it merely shows that the particular kind of map you are trying to construct cannot be a bijection. For the proof to be valid, it would need to show that any map from $mathbbS$ to $mathbbR$ is not a bijection.






            share|cite|improve this answer









            $endgroup$



            While you showed $|mathbbR| leq |mathbbS|$ properly by constructing a surjection, the problem with your proof of $|mathbbR| neq |mathbbS|$ is that it merely shows that the particular kind of map you are trying to construct cannot be a bijection. For the proof to be valid, it would need to show that any map from $mathbbS$ to $mathbbR$ is not a bijection.







            share|cite|improve this answer












            share|cite|improve this answer



            share|cite|improve this answer










            answered 7 hours ago









            MagmaMagma

            7121 silver badge8 bronze badges




            7121 silver badge8 bronze badges























                3












                $begingroup$

                As explained in the comments, what you've done is whip up a non-injective surjection from $mathbbS$ to $mathbbR$. This alone doesn't prove the statement in question, however - just because one surjection isn't injective doesn't mean that there isn't some other surjection which is injective.



                This is a fundamental difference between the infinite and finite cases. If $A$ and $B$ are finite sets and $f:Arightarrow B$ is a non-injective surjection, then $A$ does in fact have greater cardinality than $B$. But this fails for infinite sets: consider for example the map from naturals to naturals given by $$xmapsto [xover 2]$$ (where "$[cdot]$" is the floor function). This is a non-injective surjection; should we conclude that $vertmathbbNvert>vertmathbbNvert$? You can easily whip up other examples to drive the point home.



                Indeed, $mathbbR$ and $mathbbS$ have the same cardinality. Matt Samuel's comment has suggested an explanation: that the set $mathbbN^mathbbN$ of maps from naturals to naturals - or equivalently, infinite sequences of natural numbers - has cardinality at least that of $mathbbS$, and so we'll be done if we can find an injection from $mathbbN^mathbbN$ to $mathbbR$. We can do this by "coding into the binary expansion" - think about how the real number $$0.colorred001colorred00001colorred0001colorred01...$$ corresponds to the sequence $$2,4,3,1,....$$ Do you see how to turn this into an honest-to-goodness injection?




                Incidentally, while it doesn't make a difference in the context of the axiom of choice (which tells us that given a surjection $Arightarrow B$ we can find an injection $Brightarrow A$), it will ultimately be better to think of cardinality in terms of injections rather than surjections: $vert Avertgevert Bvert$ iff there is an injection $Brightarrow A$, not iff there is a surjection $Arightarrow B$. It's not immediately obvious that this is a better notion when choice fails, but it really does turn out to be the right one.






                share|cite|improve this answer









                $endgroup$












                • $begingroup$
                  Funny how we both wrote similar answers. :P
                  $endgroup$
                  – Asaf Karagila
                  7 hours ago










                • $begingroup$
                  @AsafKaragila As is our way. (Beat me by two minutes ...)
                  $endgroup$
                  – Noah Schweber
                  7 hours ago















                3












                $begingroup$

                As explained in the comments, what you've done is whip up a non-injective surjection from $mathbbS$ to $mathbbR$. This alone doesn't prove the statement in question, however - just because one surjection isn't injective doesn't mean that there isn't some other surjection which is injective.



                This is a fundamental difference between the infinite and finite cases. If $A$ and $B$ are finite sets and $f:Arightarrow B$ is a non-injective surjection, then $A$ does in fact have greater cardinality than $B$. But this fails for infinite sets: consider for example the map from naturals to naturals given by $$xmapsto [xover 2]$$ (where "$[cdot]$" is the floor function). This is a non-injective surjection; should we conclude that $vertmathbbNvert>vertmathbbNvert$? You can easily whip up other examples to drive the point home.



                Indeed, $mathbbR$ and $mathbbS$ have the same cardinality. Matt Samuel's comment has suggested an explanation: that the set $mathbbN^mathbbN$ of maps from naturals to naturals - or equivalently, infinite sequences of natural numbers - has cardinality at least that of $mathbbS$, and so we'll be done if we can find an injection from $mathbbN^mathbbN$ to $mathbbR$. We can do this by "coding into the binary expansion" - think about how the real number $$0.colorred001colorred00001colorred0001colorred01...$$ corresponds to the sequence $$2,4,3,1,....$$ Do you see how to turn this into an honest-to-goodness injection?




                Incidentally, while it doesn't make a difference in the context of the axiom of choice (which tells us that given a surjection $Arightarrow B$ we can find an injection $Brightarrow A$), it will ultimately be better to think of cardinality in terms of injections rather than surjections: $vert Avertgevert Bvert$ iff there is an injection $Brightarrow A$, not iff there is a surjection $Arightarrow B$. It's not immediately obvious that this is a better notion when choice fails, but it really does turn out to be the right one.






                share|cite|improve this answer









                $endgroup$












                • $begingroup$
                  Funny how we both wrote similar answers. :P
                  $endgroup$
                  – Asaf Karagila
                  7 hours ago










                • $begingroup$
                  @AsafKaragila As is our way. (Beat me by two minutes ...)
                  $endgroup$
                  – Noah Schweber
                  7 hours ago













                3












                3








                3





                $begingroup$

                As explained in the comments, what you've done is whip up a non-injective surjection from $mathbbS$ to $mathbbR$. This alone doesn't prove the statement in question, however - just because one surjection isn't injective doesn't mean that there isn't some other surjection which is injective.



                This is a fundamental difference between the infinite and finite cases. If $A$ and $B$ are finite sets and $f:Arightarrow B$ is a non-injective surjection, then $A$ does in fact have greater cardinality than $B$. But this fails for infinite sets: consider for example the map from naturals to naturals given by $$xmapsto [xover 2]$$ (where "$[cdot]$" is the floor function). This is a non-injective surjection; should we conclude that $vertmathbbNvert>vertmathbbNvert$? You can easily whip up other examples to drive the point home.



                Indeed, $mathbbR$ and $mathbbS$ have the same cardinality. Matt Samuel's comment has suggested an explanation: that the set $mathbbN^mathbbN$ of maps from naturals to naturals - or equivalently, infinite sequences of natural numbers - has cardinality at least that of $mathbbS$, and so we'll be done if we can find an injection from $mathbbN^mathbbN$ to $mathbbR$. We can do this by "coding into the binary expansion" - think about how the real number $$0.colorred001colorred00001colorred0001colorred01...$$ corresponds to the sequence $$2,4,3,1,....$$ Do you see how to turn this into an honest-to-goodness injection?




                Incidentally, while it doesn't make a difference in the context of the axiom of choice (which tells us that given a surjection $Arightarrow B$ we can find an injection $Brightarrow A$), it will ultimately be better to think of cardinality in terms of injections rather than surjections: $vert Avertgevert Bvert$ iff there is an injection $Brightarrow A$, not iff there is a surjection $Arightarrow B$. It's not immediately obvious that this is a better notion when choice fails, but it really does turn out to be the right one.






                share|cite|improve this answer









                $endgroup$



                As explained in the comments, what you've done is whip up a non-injective surjection from $mathbbS$ to $mathbbR$. This alone doesn't prove the statement in question, however - just because one surjection isn't injective doesn't mean that there isn't some other surjection which is injective.



                This is a fundamental difference between the infinite and finite cases. If $A$ and $B$ are finite sets and $f:Arightarrow B$ is a non-injective surjection, then $A$ does in fact have greater cardinality than $B$. But this fails for infinite sets: consider for example the map from naturals to naturals given by $$xmapsto [xover 2]$$ (where "$[cdot]$" is the floor function). This is a non-injective surjection; should we conclude that $vertmathbbNvert>vertmathbbNvert$? You can easily whip up other examples to drive the point home.



                Indeed, $mathbbR$ and $mathbbS$ have the same cardinality. Matt Samuel's comment has suggested an explanation: that the set $mathbbN^mathbbN$ of maps from naturals to naturals - or equivalently, infinite sequences of natural numbers - has cardinality at least that of $mathbbS$, and so we'll be done if we can find an injection from $mathbbN^mathbbN$ to $mathbbR$. We can do this by "coding into the binary expansion" - think about how the real number $$0.colorred001colorred00001colorred0001colorred01...$$ corresponds to the sequence $$2,4,3,1,....$$ Do you see how to turn this into an honest-to-goodness injection?




                Incidentally, while it doesn't make a difference in the context of the axiom of choice (which tells us that given a surjection $Arightarrow B$ we can find an injection $Brightarrow A$), it will ultimately be better to think of cardinality in terms of injections rather than surjections: $vert Avertgevert Bvert$ iff there is an injection $Brightarrow A$, not iff there is a surjection $Arightarrow B$. It's not immediately obvious that this is a better notion when choice fails, but it really does turn out to be the right one.







                share|cite|improve this answer












                share|cite|improve this answer



                share|cite|improve this answer










                answered 7 hours ago









                Noah SchweberNoah Schweber

                134k10 gold badges159 silver badges303 bronze badges




                134k10 gold badges159 silver badges303 bronze badges











                • $begingroup$
                  Funny how we both wrote similar answers. :P
                  $endgroup$
                  – Asaf Karagila
                  7 hours ago










                • $begingroup$
                  @AsafKaragila As is our way. (Beat me by two minutes ...)
                  $endgroup$
                  – Noah Schweber
                  7 hours ago
















                • $begingroup$
                  Funny how we both wrote similar answers. :P
                  $endgroup$
                  – Asaf Karagila
                  7 hours ago










                • $begingroup$
                  @AsafKaragila As is our way. (Beat me by two minutes ...)
                  $endgroup$
                  – Noah Schweber
                  7 hours ago















                $begingroup$
                Funny how we both wrote similar answers. :P
                $endgroup$
                – Asaf Karagila
                7 hours ago




                $begingroup$
                Funny how we both wrote similar answers. :P
                $endgroup$
                – Asaf Karagila
                7 hours ago












                $begingroup$
                @AsafKaragila As is our way. (Beat me by two minutes ...)
                $endgroup$
                – Noah Schweber
                7 hours ago




                $begingroup$
                @AsafKaragila As is our way. (Beat me by two minutes ...)
                $endgroup$
                – Noah Schweber
                7 hours ago











                2












                $begingroup$

                Consider the function $FcolonBbbNto N$ defined by: $$F(n)=begincases 0 & n=0\ n-1 & n>0endcases$$
                by your argument, since this function is not a bijection (clearly $F(0)=F(1)$ so it is not injective), $Bbb N$ is not the same size as $Bbb N$.




                What just happened? You've proved, as I did above, there is a surjective function which is not injective. But that doesn't prevents a different function to be a bijection.



                Cantor's diagonal works by showing that any function is not surjective, and therefore there are no bijections. But you've only showed that one function is not injective, and concluded that there are no bijections.



                You have to remember: Infinite sets are very different from finite sets. Just because one function is a surjection which is not a bijection doesn't mean there a different one is not.



                Here's an injective function from the rearrangements into the reals:



                Let $fcolonBbbNto N$ be the permutation describing the rearrangement. Namely, $f(n)=k$ if and only if the $k$th summand is $frac1n$ (up to a sign, if you prefer). Map the rearrangement given by $f$ to the real number given by: $$sum_ninBbb Nfrac23^2^ncdot 3^f(n)$$



                This sum converges, and you can quite easily show that if $fneq g$, then there is some $n$ such that $f(n)neq g(n)$ and therefore one of the digits in the trenary expansion of the two reals is different, and since those are only $0$ or $2$ this means that the real numbers are different. Therefore the map from rearrangements into the real numbers given above is injective.



                On the other hand, you can easily show that there are at least $2^aleph_0$ rearrangements, but I will leave this to you as an exercise.






                share|cite|improve this answer









                $endgroup$

















                  2












                  $begingroup$

                  Consider the function $FcolonBbbNto N$ defined by: $$F(n)=begincases 0 & n=0\ n-1 & n>0endcases$$
                  by your argument, since this function is not a bijection (clearly $F(0)=F(1)$ so it is not injective), $Bbb N$ is not the same size as $Bbb N$.




                  What just happened? You've proved, as I did above, there is a surjective function which is not injective. But that doesn't prevents a different function to be a bijection.



                  Cantor's diagonal works by showing that any function is not surjective, and therefore there are no bijections. But you've only showed that one function is not injective, and concluded that there are no bijections.



                  You have to remember: Infinite sets are very different from finite sets. Just because one function is a surjection which is not a bijection doesn't mean there a different one is not.



                  Here's an injective function from the rearrangements into the reals:



                  Let $fcolonBbbNto N$ be the permutation describing the rearrangement. Namely, $f(n)=k$ if and only if the $k$th summand is $frac1n$ (up to a sign, if you prefer). Map the rearrangement given by $f$ to the real number given by: $$sum_ninBbb Nfrac23^2^ncdot 3^f(n)$$



                  This sum converges, and you can quite easily show that if $fneq g$, then there is some $n$ such that $f(n)neq g(n)$ and therefore one of the digits in the trenary expansion of the two reals is different, and since those are only $0$ or $2$ this means that the real numbers are different. Therefore the map from rearrangements into the real numbers given above is injective.



                  On the other hand, you can easily show that there are at least $2^aleph_0$ rearrangements, but I will leave this to you as an exercise.






                  share|cite|improve this answer









                  $endgroup$















                    2












                    2








                    2





                    $begingroup$

                    Consider the function $FcolonBbbNto N$ defined by: $$F(n)=begincases 0 & n=0\ n-1 & n>0endcases$$
                    by your argument, since this function is not a bijection (clearly $F(0)=F(1)$ so it is not injective), $Bbb N$ is not the same size as $Bbb N$.




                    What just happened? You've proved, as I did above, there is a surjective function which is not injective. But that doesn't prevents a different function to be a bijection.



                    Cantor's diagonal works by showing that any function is not surjective, and therefore there are no bijections. But you've only showed that one function is not injective, and concluded that there are no bijections.



                    You have to remember: Infinite sets are very different from finite sets. Just because one function is a surjection which is not a bijection doesn't mean there a different one is not.



                    Here's an injective function from the rearrangements into the reals:



                    Let $fcolonBbbNto N$ be the permutation describing the rearrangement. Namely, $f(n)=k$ if and only if the $k$th summand is $frac1n$ (up to a sign, if you prefer). Map the rearrangement given by $f$ to the real number given by: $$sum_ninBbb Nfrac23^2^ncdot 3^f(n)$$



                    This sum converges, and you can quite easily show that if $fneq g$, then there is some $n$ such that $f(n)neq g(n)$ and therefore one of the digits in the trenary expansion of the two reals is different, and since those are only $0$ or $2$ this means that the real numbers are different. Therefore the map from rearrangements into the real numbers given above is injective.



                    On the other hand, you can easily show that there are at least $2^aleph_0$ rearrangements, but I will leave this to you as an exercise.






                    share|cite|improve this answer









                    $endgroup$



                    Consider the function $FcolonBbbNto N$ defined by: $$F(n)=begincases 0 & n=0\ n-1 & n>0endcases$$
                    by your argument, since this function is not a bijection (clearly $F(0)=F(1)$ so it is not injective), $Bbb N$ is not the same size as $Bbb N$.




                    What just happened? You've proved, as I did above, there is a surjective function which is not injective. But that doesn't prevents a different function to be a bijection.



                    Cantor's diagonal works by showing that any function is not surjective, and therefore there are no bijections. But you've only showed that one function is not injective, and concluded that there are no bijections.



                    You have to remember: Infinite sets are very different from finite sets. Just because one function is a surjection which is not a bijection doesn't mean there a different one is not.



                    Here's an injective function from the rearrangements into the reals:



                    Let $fcolonBbbNto N$ be the permutation describing the rearrangement. Namely, $f(n)=k$ if and only if the $k$th summand is $frac1n$ (up to a sign, if you prefer). Map the rearrangement given by $f$ to the real number given by: $$sum_ninBbb Nfrac23^2^ncdot 3^f(n)$$



                    This sum converges, and you can quite easily show that if $fneq g$, then there is some $n$ such that $f(n)neq g(n)$ and therefore one of the digits in the trenary expansion of the two reals is different, and since those are only $0$ or $2$ this means that the real numbers are different. Therefore the map from rearrangements into the real numbers given above is injective.



                    On the other hand, you can easily show that there are at least $2^aleph_0$ rearrangements, but I will leave this to you as an exercise.







                    share|cite|improve this answer












                    share|cite|improve this answer



                    share|cite|improve this answer










                    answered 7 hours ago









                    Asaf KaragilaAsaf Karagila

                    313k34 gold badges451 silver badges784 bronze badges




                    313k34 gold badges451 silver badges784 bronze badges





















                        2












                        $begingroup$

                        You have shown the cardinality of the real numbers is a lower bound by constructing a surjective map. You can't conclude from the fact that it is not injective that there are more rearrangements, because in order to do that you'd need to be able to show that every map is not injective.



                        In fact, the cardinality of the set of functions from $mathbb N$ to $mathbb N$ is an upper bound, and that is the same as that of the real numbers. So in fact there are equally many arrangements as real numbers.



                        In Cantor's diagonal argument, we are given an arbitrary function from the naturals to the real numbers, and the argument provides an algorithm for finding a number that the function doesn't hit. This works because it's not just one function this works for, it is every function. Therefore no function from the naturals to the real numbers is surjective.






                        share|cite|improve this answer











                        $endgroup$

















                          2












                          $begingroup$

                          You have shown the cardinality of the real numbers is a lower bound by constructing a surjective map. You can't conclude from the fact that it is not injective that there are more rearrangements, because in order to do that you'd need to be able to show that every map is not injective.



                          In fact, the cardinality of the set of functions from $mathbb N$ to $mathbb N$ is an upper bound, and that is the same as that of the real numbers. So in fact there are equally many arrangements as real numbers.



                          In Cantor's diagonal argument, we are given an arbitrary function from the naturals to the real numbers, and the argument provides an algorithm for finding a number that the function doesn't hit. This works because it's not just one function this works for, it is every function. Therefore no function from the naturals to the real numbers is surjective.






                          share|cite|improve this answer











                          $endgroup$















                            2












                            2








                            2





                            $begingroup$

                            You have shown the cardinality of the real numbers is a lower bound by constructing a surjective map. You can't conclude from the fact that it is not injective that there are more rearrangements, because in order to do that you'd need to be able to show that every map is not injective.



                            In fact, the cardinality of the set of functions from $mathbb N$ to $mathbb N$ is an upper bound, and that is the same as that of the real numbers. So in fact there are equally many arrangements as real numbers.



                            In Cantor's diagonal argument, we are given an arbitrary function from the naturals to the real numbers, and the argument provides an algorithm for finding a number that the function doesn't hit. This works because it's not just one function this works for, it is every function. Therefore no function from the naturals to the real numbers is surjective.






                            share|cite|improve this answer











                            $endgroup$



                            You have shown the cardinality of the real numbers is a lower bound by constructing a surjective map. You can't conclude from the fact that it is not injective that there are more rearrangements, because in order to do that you'd need to be able to show that every map is not injective.



                            In fact, the cardinality of the set of functions from $mathbb N$ to $mathbb N$ is an upper bound, and that is the same as that of the real numbers. So in fact there are equally many arrangements as real numbers.



                            In Cantor's diagonal argument, we are given an arbitrary function from the naturals to the real numbers, and the argument provides an algorithm for finding a number that the function doesn't hit. This works because it's not just one function this works for, it is every function. Therefore no function from the naturals to the real numbers is surjective.







                            share|cite|improve this answer














                            share|cite|improve this answer



                            share|cite|improve this answer








                            edited 5 hours ago









                            Noah Schweber

                            134k10 gold badges159 silver badges303 bronze badges




                            134k10 gold badges159 silver badges303 bronze badges










                            answered 7 hours ago









                            Matt SamuelMatt Samuel

                            40.5k6 gold badges38 silver badges71 bronze badges




                            40.5k6 gold badges38 silver badges71 bronze badges



























                                draft saved

                                draft discarded
















































                                Thanks for contributing an answer to Mathematics Stack Exchange!


                                • Please be sure to answer the question. Provide details and share your research!

                                But avoid


                                • Asking for help, clarification, or responding to other answers.

                                • Making statements based on opinion; back them up with references or personal experience.

                                Use MathJax to format equations. MathJax reference.


                                To learn more, see our tips on writing great answers.




                                draft saved


                                draft discarded














                                StackExchange.ready(
                                function ()
                                StackExchange.openid.initPostLogin('.new-post-login', 'https%3a%2f%2fmath.stackexchange.com%2fquestions%2f3291289%2ftrying-to-find-a-flaw-in-my-proof-that-there-are-more-rearrangements-of-an-infin%23new-answer', 'question_page');

                                );

                                Post as a guest















                                Required, but never shown





















































                                Required, but never shown














                                Required, but never shown












                                Required, but never shown







                                Required, but never shown

































                                Required, but never shown














                                Required, but never shown












                                Required, but never shown







                                Required, but never shown







                                Popular posts from this blog

                                Invision Community Contents History See also References External links Navigation menuProprietaryinvisioncommunity.comIPS Community ForumsIPS Community Forumsthis blog entry"License Changes, IP.Board 3.4, and the Future""Interview -- Matt Mecham of Ibforums""CEO Invision Power Board, Matt Mecham Is a Liar, Thief!"IPB License Explanation 1.3, 1.3.1, 2.0, and 2.1ArchivedSecurity Fixes, Updates And Enhancements For IPB 1.3.1Archived"New Demo Accounts - Invision Power Services"the original"New Default Skin"the original"Invision Power Board 3.0.0 and Applications Released"the original"Archived copy"the original"Perpetual licenses being done away with""Release Notes - Invision Power Services""Introducing: IPS Community Suite 4!"Invision Community Release Notes

                                Canceling a color specificationRandomly assigning color to Graphics3D objects?Default color for Filling in Mathematica 9Coloring specific elements of sets with a prime modified order in an array plotHow to pick a color differing significantly from the colors already in a given color list?Detection of the text colorColor numbers based on their valueCan color schemes for use with ColorData include opacity specification?My dynamic color schemes

                                Tom Holland Mục lục Đầu đời và giáo dục | Sự nghiệp | Cuộc sống cá nhân | Phim tham gia | Giải thưởng và đề cử | Chú thích | Liên kết ngoài | Trình đơn chuyển hướngProfile“Person Details for Thomas Stanley Holland, "England and Wales Birth Registration Index, 1837-2008" — FamilySearch.org”"Meet Tom Holland... the 16-year-old star of The Impossible""Schoolboy actor Tom Holland finds himself in Oscar contention for role in tsunami drama"“Naomi Watts on the Prince William and Harry's reaction to her film about the late Princess Diana”lưu trữ"Holland and Pflueger Are West End's Two New 'Billy Elliots'""I'm so envious of my son, the movie star! British writer Dominic Holland's spent 20 years trying to crack Hollywood - but he's been beaten to it by a very unlikely rival"“Richard and Margaret Povey of Jersey, Channel Islands, UK: Information about Thomas Stanley Holland”"Tom Holland to play Billy Elliot""New Billy Elliot leaving the garage"Billy Elliot the Musical - Tom Holland - Billy"A Tale of four Billys: Tom Holland""The Feel Good Factor""Thames Christian College schoolboys join Myleene Klass for The Feelgood Factor""Government launches £600,000 arts bursaries pilot""BILLY's Chapman, Holland, Gardner & Jackson-Keen Visit Prime Minister""Elton John 'blown away' by Billy Elliot fifth birthday" (video with John's interview and fragments of Holland's performance)"First News interviews Arrietty's Tom Holland"“33rd Critics' Circle Film Awards winners”“National Board of Review Current Awards”Bản gốc"Ron Howard Whaling Tale 'In The Heart Of The Sea' Casts Tom Holland"“'Spider-Man' Finds Tom Holland to Star as New Web-Slinger”lưu trữ“Captain America: Civil War (2016)”“Film Review: ‘Captain America: Civil War’”lưu trữ“‘Captain America: Civil War’ review: Choose your own avenger”lưu trữ“The Lost City of Z reviews”“Sony Pictures and Marvel Studios Find Their 'Spider-Man' Star and Director”“‘Mary Magdalene’, ‘Current War’ & ‘Wind River’ Get 2017 Release Dates From Weinstein”“Lionsgate Unleashing Daisy Ridley & Tom Holland Starrer ‘Chaos Walking’ In Cannes”“PTA's 'Master' Leads Chicago Film Critics Nominations, UPDATED: Houston and Indiana Critics Nominations”“Nominaciones Goya 2013 Telecinco Cinema – ENG”“Jameson Empire Film Awards: Martin Freeman wins best actor for performance in The Hobbit”“34th Annual Young Artist Awards”Bản gốc“Teen Choice Awards 2016—Captain America: Civil War Leads Second Wave of Nominations”“BAFTA Film Award Nominations: ‘La La Land’ Leads Race”“Saturn Awards Nominations 2017: 'Rogue One,' 'Walking Dead' Lead”Tom HollandTom HollandTom HollandTom Hollandmedia.gettyimages.comWorldCat Identities300279794no20130442900000 0004 0355 42791085670554170004732cb16706349t(data)XX5557367